Proving that (6k+1) is Closed Under Multiplication

Click For Summary
The discussion focuses on proving that the set defined by the expression (6k + 1) is closed under multiplication. Participants explore the multiplication of two elements in this form, (6k + 1)(6n + 1), and seek to confirm that the result can also be expressed as (6m + 1) for some integer m. There is a mention of showing that the set (6k + 5) contains primes, including the example of 11, and the challenge of demonstrating the existence of infinitely many primes in that set. The conversation emphasizes the importance of correctly defining variables to avoid confusion in mathematical proofs. Overall, the thread highlights the complexities of number theory and the collaborative effort to understand these concepts.
forty
Messages
132
Reaction score
0
Show the progression (6k +1) (k is an integer) is closed under multiplication:

Firstly I should check that I remember what this means... If it is closed when you multiply any 2 elements together you get an element that is in the set?

So for this I thought just show (6k+1)(6n+1), where k and n are integers, is of the form (6k+1) but I don't think that works and can't think of any other ways to do it.

Show that the progression (6k+5) contains a prime. Then show it contains infinitely many primes:

Show that it contains a prime 6(1)+5=11 (is that part really that simple :S?) As for showing it has infinitely many...

Any help with this would be greatly appreciated. I'm actually on exchange at Lund Uni in Sweden (I come from Melbourne Uni, Australia) and I'll be stuffed if I can understand my lecturer (on Number Theory) so until I can get my hands on a copy of the textbook this stuff is doing my head in!
 
Physics news on Phys.org
forty said:
So for this I thought just show (6k+1)(6n+1), where k and n are integers, is of the form (6m+1) but I don't think that works and can't think of any other ways to do it.
I've changed your second usage of k to m; you shouldn't use the same variable for multiple purposes, you can confuse yourself.

To show something is of the form 6m+1, you just need to find m, right?
 
(6k+1)(6n+1)
6(6kn+k+n)+1
m = (6kn+k+n)

is that what you mean?
 
That sounds good. Now all that's left is to check that m is the right kind of object.

Remember, you wanted the form "6m+1 where m is an integer" -- so from "6m+1" we've narrowed things down to one possibility for m, so now we just have to plug into the other condition to see if it works.

(Yes, this check is trivial in this case. But it can be less trival in other problems)
 
Question: A clock's minute hand has length 4 and its hour hand has length 3. What is the distance between the tips at the moment when it is increasing most rapidly?(Putnam Exam Question) Answer: Making assumption that both the hands moves at constant angular velocities, the answer is ## \sqrt{7} .## But don't you think this assumption is somewhat doubtful and wrong?

Similar threads

  • · Replies 16 ·
Replies
16
Views
3K
  • · Replies 7 ·
Replies
7
Views
3K
  • · Replies 3 ·
Replies
3
Views
3K
  • · Replies 3 ·
Replies
3
Views
5K
  • · Replies 1 ·
Replies
1
Views
1K
Replies
12
Views
3K
Replies
4
Views
3K
Replies
2
Views
1K
  • · Replies 3 ·
Replies
3
Views
2K
  • · Replies 2 ·
Replies
2
Views
2K